Question

Given a total cost function of TC = 120 + 2Q^2 determine the marginal cost function...

Given a total cost function of TC = 120 + 2Q^2 determine the marginal cost function which is obtained by differentiating the total cost function. What will be the total cost and marginal cost at an output of 10 units?

Homework Answers

Answer #1

The total cost function is as follows -

TC = 120 + 2Q2

Determine the marginal cost function -

MC = dTC/dQ

MC = d(120 + 2Q2)/dQ

MC = 4Q

The marginal cost function is 4Q.

Calculate the total cost when output is 10 units -

Total cost = 120 + 2Q2 = 120 + [2 * (10)2] = 120 + 200 = 320

Thus,

The total cost when output is 10 units is $320.

Calculate the marginal cost when 10 units are produced -

MC = 4Q = 4 * 10 = 40

Thus,

The marginal cost when 10 units are produced is $40.

Know the answer?
Your Answer:

Post as a guest

Your Name:

What's your source?

Earn Coins

Coins can be redeemed for fabulous gifts.

Not the answer you're looking for?
Ask your own homework help question
Similar Questions
Given the total cost equation: TC=144 + 2Q + Q^2 what is the average cost when...
Given the total cost equation: TC=144 + 2Q + Q^2 what is the average cost when the firm produces at the level of Q that yields the minimum average cost?
Given TC = 40 + 2q + 0.1q2 + 0.02q3. a.What is this firm’s total fixed...
Given TC = 40 + 2q + 0.1q2 + 0.02q3. a.What is this firm’s total fixed cost? b.What is the equation for this firm’s average variable cost? c.If this firm’s profit maximizing level of output is 10 units, would this firm shutdown if it sells its product at $7 per unit? Explain briefly.
A firms demand function for a good is given by P = 107-2Q and their total...
A firms demand function for a good is given by P = 107-2Q and their total cost function is given by TC = 200+3Q. (using for 6.1 to 6.4) 6.1 Obtain an expression for total revenue (price X quantity) in terms of Q 6.2 For what values of Q does the firm breakeven? 6.3 Illustrate the answer to (ii) using sketches of the total cost function, the total revenue function and the profit function 6.4 From the graph estimate the...
A firm's total cost function is given by the equation: TC = 4000 + 5Q +...
A firm's total cost function is given by the equation: TC = 4000 + 5Q + 10Q2. (1) Write an expression for each of the following cost concepts: a. Total Fixed Cost b. Average Fixed Cost c. Total Variable Cost d. Average Variable Cost e. Average Total Cost f. Marginal Cost (2) Determine the quantity that minimizes average total cost and minimizing average variable cost. (3) Why does its average variable cost curve achieve its minimum at a lower level...
A perfectly competitive firm in the short run has Total Cost and Marginal Cost functions given...
A perfectly competitive firm in the short run has Total Cost and Marginal Cost functions given by TC(Q)=9+Q+Q2 and MC(Q)=1+2Q, respectively. The firm faces a price of P=$17. Determine the output that the firm will produce and the profit. Show the solution graphically.
3. Suppose that a price-searcher monopolist had a total cost function given by: TC= 20 +...
3. Suppose that a price-searcher monopolist had a total cost function given by: TC= 20 + 2Q +0.25Q2. The demand for the price searcher's product is given by: QD= 100 -5P. Calculate the price the monopolist will charge. (Do not include a dollar sign in your response. Round to the nearest two decimals.) 4. Suppose that a price-searcher monopolist had a total cost function given by: TC= 20 + 2Q +0.25Q2. The demand for the price searcher's product is given...
1. The short-run cost function of a company is given by the equation TC=200+55q, where TC...
1. The short-run cost function of a company is given by the equation TC=200+55q, where TC is the total cost and q is the total quantity of output. If the company produced 100 units of the good, what is the average fixed cost? A. 200 B. 55 C. 2 D. 100
if the marginal cost of a firm is MC = 9q^2 + 2q +1 and the...
if the marginal cost of a firm is MC = 9q^2 + 2q +1 and the marginal revenue MR = 60-q. Given that total cost is 3390 when q -10 (a) Derive an expression for total cost (b) Derive the expression for total revenue (c) Using the results from (a) and (b) find the total profit function
A perfectly competitive firm has the following total cost and marginal cost functions:      TC =...
A perfectly competitive firm has the following total cost and marginal cost functions:      TC = 100 + 10q – q2 + (1/3)q3      MC = q2 – 2q +10      a)    For quantities from 0 to 10 determine: TC, TFC, TVC, and MC. b)    For quantities from 0 to 10 determine: ATC, AFC, and AVC. c)    Assume P (MR) equals 45. For quantities from 0 to 10 determine: TR and profit. d)    At what quantity is profit maximized?...
the demand equiation of a goof is given by P+2Q=20 abd the total cost function is...
the demand equiation of a goof is given by P+2Q=20 abd the total cost function is Q3-8Q^2+20Q+2 a) find the level of output that maximizes total revenue b) find the maximum profit and the value of Q at which it is achieved. verify that, at this value of Q, MR=MC